Help with 2 Problems in Differential Equations

  • Thread starter Thread starter ballajr
  • Start date Start date
Click For Summary
The discussion addresses two problems in differential equations. The first problem involves finding the general solution to a differential equation, with suggestions to use the method of Undetermined Coefficients or the annihilator method. The second problem requires proving or disproving the existence of constants A and B in a specific equation, with a correction noted regarding the equation's form. It is mentioned that complex solutions may allow for valid values of A and B. Overall, participants are encouraged to share their attempts and clarify any confusion in their calculations.
ballajr
Messages
8
Reaction score
0

Homework Statement



1) Find the General Solution to:
(D3 - D2 + D - I)[y] = t5 + 1

2) Prove or disprove that there are two constants A and B such that:
t2D - tD - 8I = (tD + AI)(tD + BI)



Homework Equations




The Attempt at a Solution



1) I can't figure out how to attempt this one. Doesn't make sense to me.
2) I should FOIL out the RHS of the equation, but I did that on paper and it didn't make too much sense to me.
 
Physics news on Phys.org
ballajr said:
1) I can't figure out how to attempt this one. Doesn't make sense to me.

There are two methods that immediately come to mind:

(1)Use the method of Undetermined Coefficients

(2)Use the annihilator method

2) I should FOIL out the RHS of the equation, but I did that on paper and it didn't make too much sense to me.

Show us what you've got, and keep in mind that to calculate something like D(tD) you need to use the product rule.
 
There was a typo in #2:

2) Prove or disprove that there are two constants A and B such that:
t2D2 - tD - 8I = (tD + AI)(tD + BI)
 
There are solutions for A and B if you permit complex solutions.
 
Question: A clock's minute hand has length 4 and its hour hand has length 3. What is the distance between the tips at the moment when it is increasing most rapidly?(Putnam Exam Question) Answer: Making assumption that both the hands moves at constant angular velocities, the answer is ## \sqrt{7} .## But don't you think this assumption is somewhat doubtful and wrong?

Similar threads

Replies
11
Views
2K
  • · Replies 4 ·
Replies
4
Views
2K
  • · Replies 10 ·
Replies
10
Views
2K
Replies
2
Views
2K
  • · Replies 7 ·
Replies
7
Views
2K
  • · Replies 3 ·
Replies
3
Views
1K
  • · Replies 5 ·
Replies
5
Views
2K
  • · Replies 3 ·
Replies
3
Views
5K
  • · Replies 1 ·
Replies
1
Views
1K
  • · Replies 10 ·
Replies
10
Views
3K